2022 年江苏高考数学(新高考全国 I)试卷

听很多人说这份卷子十分难,于是打算简单做一下吧。

单项选择题

单项选择一共 8 条,一条 5 分,一共是 40 分。错一条的代价实在是太大了,所以还是小心为好。

我给出的答案为 (D)(D)(B)(C)(D)(A)(C)(C)。

Problem 1

如果集合 \(M = \{x: \sqrt x < 4\}, N = \{x: 3x \geq 1\}\),求集合 \(M \cap N\)

  1. \(\{x: 0\leq x < 2\}\)
  2. \(\left\{x: \dfrac13 \leq x < 2\right\}\)
  3. \(\{x: 3 \leq x < 16\}\)
  4. \(\left\{x: \dfrac13 \leq x < 16\right\}\)

显然 \(M = (0, 16), N = \left[\dfrac13, +\infty\right)\),答案为 (D)。

Problem 2

如果 \(\mathop{\rm i}(1 - z) = 1\),求 \(z + \overline z\) 的值。

  1. \(-2\)
  2. \(-1\)
  3. \(1\)
  4. \(2\)

假设 \(z := a + b\mathop{\rm i}(a, b \in \mathbb R)\),那么:

\[ 1 = \mathop{\rm i}(1 - z) = \mathop{\rm i}((1 - a) - b\mathop{\rm i}) = b + (1 - a)\mathop{\rm i} \]

得到 \(z = 1 + \mathop{\rm i}\),所以 \(z + \overline z = 2\Re(z) = 2\),答案为 (D)。

Problem 3

\(\triangle ABC\) 中,点 \(D\) 在边 \(AB\) 上,\(BD = 2DA\),记 \(\vec{CA} = \boldsymbol{m}, \vec{CD} = \boldsymbol{n}\),那么:

  1. \(\vec{CB} = 3\boldsymbol{m} - 2\boldsymbol{n}\)
  2. \(\vec{CB} = -2\boldsymbol{m} + 3\boldsymbol{n}\)
  3. \(\vec{CB} = 3\boldsymbol{m} + 2\boldsymbol{n}\)
  4. \(\vec{CB} = 2\boldsymbol{m} + 3\boldsymbol{n}\)

显然有 \(\boldsymbol{n} = \vec{CD} = \vec{CB} + \vec{BD}\),也就是说 \(\vec{BD} = \boldsymbol{n} - \vec{CB}\)

同理我们还能得到 \(\vec{AD} = \vec{CD} - \vec{CA} = \boldsymbol{n} - \boldsymbol{m}\)

根据 \(2\vec{AD} + \vec{BD} = \boldsymbol{0}\) 得到:

\[ \boldsymbol{n} - \vec{CB} + 2(\boldsymbol{n} - \boldsymbol{m}) = \boldsymbol{0} \]

也就是:

\[ \vec{CB} = -2\boldsymbol{m} + 3\boldsymbol{n} \]

答案为 (B)。

Problem 4

一道应用题。大意为,一个棱台水库,水位为海拔 \(148.5\) 米的时候,水面面积为 \(140.0\) 平方千米。水位为海拔 \(157.5\) 米的时候,水面面积为 \(180.0\) 平方千米。求解水库水位从 \(148.5\) 米上升到 \(157.5\) 米时增加的水量。

  1. \(1.0 \times 10^9\) 立方米
  2. \(1.2 \times 10^9\) 立方米
  3. \(1.4 \times 10^9\) 立方米
  4. \(1.6 \times 10^9\) 立方米

简单的棱台体积计算题,注意单位换算:

\[ \begin{aligned} V &= \frac13 h(S_1 + S_2 + \sqrt{S_1S_2}) \\ &= \frac13 \times (157.5 - 148.5) \times(140 + 180 + \sqrt{140 \times 180}) \times 10^6 \\ &= 6(3\sqrt 7 + 16) \times 10^7 \\ &\approx 1.4 \times 10^9 ({\rm m}^3) \end{aligned} \]

答案为 (C)。

Problem 5

\(2\)\(8\) 总共 \(7\) 个整数中随机取两个不同的数,求两数互质的概率。

  1. \(1/6\)
  2. \(1/3\)
  3. \(1/2\)
  4. \(2/3\)

互质的整数对包括:

\[ \begin{aligned} &(2, 3), (2, 5), (2, 7) \\ &(3, 4), (3, 5), (3, 7), (3, 8) \\ &(4, 5), (4, 7) \\ &(5, 6), (5, 7), (5, 8) \\ &(6, 7) \\ &(7, 8) \\ \end{aligned} \]

所以概率为:

\[ p = \frac{14}{C_7^6} = \frac23 \]

答案为 (D)。

Problem 6

函数 \(f(x) = \sin\left(\omega x + \dfrac\pi 4\right) + b(\omega > 0)\) 的最小正周期为 \(T\),如果 \(\dfrac{2\pi}{3} < T < \pi\),而且曲线 \(y = f(x)\) 关于点 \(\left(\dfrac{3\pi}{2}, 2\right)\) 对称,求 \(f\left(\dfrac\pi 2\right)\) 的值。

  1. \(1\)
  2. \(3/2\)
  3. \(5/2\)
  4. \(3\)

根据曲线 \(y = f(x)\) 关于点 \(\left(\dfrac{3\pi}{2}, 2\right)\) 对称,能得到 \(f\left(\dfrac{3\pi}{2}\right) = 2\),以及 \(b = 0\),也就是说:

\[ \begin{aligned} &\sin\left(\omega \dfrac{3\pi}{2} + \dfrac\pi 4\right) = 0 \\ \Rightarrow &\omega \dfrac{3\pi}{2} + \dfrac\pi 4 = k\pi \\ \Rightarrow &\omega = \frac23\left(k - \frac14\right), k \in \mathbb Z \end{aligned} \]

根据最小正周期的范围,得到:

\[ T = \frac{2\pi}{\omega} \in \left(\dfrac{2\pi}{3}, \pi\right) \Rightarrow \omega \in (2, 3) \]

进而得知 \(k = 4\),也就是说 \(\omega = \dfrac52\),那么:

\[ f\left(\dfrac\pi 2\right) = \sin\left(\dfrac52 \cdot \dfrac{\pi}{2} + \dfrac\pi 4\right) + 2 = 1 \]

答案为 (A)。

Problem 7

已知 \(a = 0.1e^{0.1}, b = \dfrac19, c = -\ln 0.9\),则:

  1. \(a < b < c\)
  2. \(c < b < a\)
  3. \(c < a < b\)
  4. \(a < c < b\)

考虑函数 \(f(x) = xe^x + \ln(1 - x), x \geq 0\),有:

\[ f'(x) = (x + 1)e^x - \frac{1}{1 - x} > (x + 1)^2 - \frac{1}{1 - x} = -\frac{x(x^2 + x - 1)}{1 - x} \]

导函数保证了函数至少在区间 \(\left(0, \dfrac{-1 + \sqrt5}{2}\right)\) 上递增,而 \(\dfrac1{10}\) 属于这个区间,所以:

\[ f(0.1) = 0.1e^{0.1} - \ln\frac{10}{9} > f(0) = 0 \]

所以 \(a > c\)

再考虑函数 \(g(x) = \ln(1 + x) - \dfrac{x}{1 + x}, x \geq 0\),有:

\[ g'(x) = \frac{1}{1 + x} - \frac{1}{(1 + x)^2} = \frac{x}{(1 + x)^2} > 0 \]

从而:

\[ g\left(\frac19\right) = \ln\frac{10}{9} - \frac{1}{10} > 0 \]

这说明:

\[ \ln\frac{10}{9} > \frac{1}{10} \Rightarrow \frac{10}{9} > e^{0.1} \Rightarrow \frac{1}{9} > 0.1e^{0.1} \]

也就是 \(b > a\),从而答案为 (C)。

Problem 8

已知正四棱锥的侧棱长为 \(l\),其各顶点都在同一个球面上,若该球体积为 \(36\pi\),且 \(3\leq l\leq 3\sqrt 3\),求该正四棱锥体积的取值范围。

  1. \(\left[18, \dfrac{81}{4}\right]\)
  2. \(\left[\dfrac{27}{4}, \dfrac{81}{4}\right]\)
  3. \(\left[\dfrac{27}{4}, \dfrac{64}{3}\right]\)
  4. \(\left[18, 27\right]\)

假设球的半径为 \(R\),显然有 \(V = \dfrac43\pi R^3 = 36\pi\),得到 \(R = 3\)

假设正四棱锥 \(O-ABCD\) 底面中心为 \(P\),其外接球球心为 \(Q\)。显然我们得到一个直角三角形 \(OPA\),而球心 \(Q\) 在直线 \(PO\) 上。令有向线段 \(QO\) 长度为 \(3\),记有向线段 \(QP\) 长度为 \(x \in (-3, 3)\)。显然:

\[ \begin{aligned} l^2 &= OA^2 = PO^2 + PA^2 \\ &= (QO - QP)^2 + (QA^2 - QP^2) \\ &= (3 - x)^2 + (9 - x^2) \\ &= 18 - 6x \in [9, 27] \end{aligned} \]

也就得到了 \(x \in \left[-\dfrac32, \dfrac32\right]\)。从而:

\[ V_{O-ABCD} = \frac13 |PO| \cdot 2PA^2 = \frac23(3 - x)(9 - x^2) \]

不难得到函数 \(f(x) = \dfrac23(3 - x)(9 - x^2)\) 在区间 \(\left[-\dfrac32, -1\right]\) 上递增,在区间 \(\left[-1, \dfrac32\right]\) 上递减,计算得到:

\[ f\left(-\dfrac32\right) = \frac{81}{4}, f\left(\dfrac32\right) = \frac{27}{4}, f(-1) = \frac{64}{3} \]

体积的取值范围应当为 \(\left[\dfrac{27}{4}, \dfrac{64}{3}\right]\),答案为 (C)。

不定项选择题

不定项选择一共 4 条,一条 5 分,一共是 20 分。

我给出的答案为 (ABD)(AC)(BCD)(BC)。

Problem 9

对于正方体 \(ABCD-A_1B_1C_1D_1\),下列说法中正确的有:

  1. 直线 \(BC_1\)\(DA_1\) 所成的角为直角
  2. 直线 \(BC_1\)\(CA_1\) 所成的角为直角
  3. 直线 \(BC_1\) 与平面 \(BB_1D_1D\) 所成的角为 \(45^\circ\)
  4. 直线 \(BC_1\) 与平面 \(ABCD\) 所成的角为 \(45^\circ\)

根据直线 \(BC_1\) 垂直于平面 \(A_1B_1CD\) 就能得到 (A)(B) 均正确。

对于 (C),显然有直线 \(A_1C_1\) 垂直于平面 \(BB_1D_1D\),而 \(\angle A_1C_1B = 60^\circ\),该选项叙述错误。

对于 (D),显然有直线 \(CC_1\) 垂直于平面 \(ABCD\),而 \(\angle CC_1B = 45^\circ\),该选项叙述正确。

所以答案为 (ABD)。

Problem 10

已知函数 \(f(x) = x^3 - x + 1\),下列说法中正确的有:

  1. \(f(x)\) 有两个极值点
  2. \(f(x)\) 有三个零点
  3. \((0, 1)\) 为曲线 \(y = f(x)\) 的对称中心
  4. 直线 \(y = 2x\) 为曲线 \(y = f(x)\) 的切线

首先:

\[ f'(x) = 3x^2 - 1 \]

得到 \(f(x)\)\(\left(-\infty, -\dfrac{\sqrt 3}{3}\right)\) 上递增,在 \(\left(-\dfrac{\sqrt 3}{3}, \dfrac{\sqrt 3}{3}\right)\) 上递减,在 \(\left(\dfrac{\sqrt 3}{3}, +\infty\right)\) 上递增。

并且计算出:

\[ \begin{aligned} f\left(-\frac{\sqrt3}{3}\right) &= \frac{2\sqrt 3}{9} + 1 > 0 \\ f\left(\frac{\sqrt3}{3}\right) &= -\frac{2\sqrt 3}{9} + 1 > 0 \\ \end{aligned} \]

所以 \(f(x)\) 有两个极值点,一个零点,选项 (A) 正确,选项 (B) 错误。

进一步,我们我们有:

\[ f(x) + f(-x) = (x^3 - x + 1) + (-x^3 + x + 1) = 2, x \in \mathbb R \]

这说明选项 (C) 正确。

如果选项 (D) 正确,假设其为 \(x = x_0\) 处的切线,根据 \(f'(x_0) = 3x_0^2 - 1 = 2\) 得到 \(x_0 = \pm 1\)

此外,我们知道曲线 \(y = f(x)\)\(x = x_0\) 处切线方程为:

\[ y = f'(x_0)(x - x_0) + f(x_0) = f'(x_0)x - x_0f'(x_0) + f(x_0) = 2x \]

那么我们得到:

\[ -x_0f'(x_0) + f(x_0) = 0 \Rightarrow 2x_0 = f(x_0) = x_0^3 - x_0 + 1 \]

\(x_0 = \pm 1\) 代入验证,发现均不成立,所以选项 (D) 错误。

答案为 (AC)。

Problem 11

已知 \(O\) 为坐标原点,点 \(A(1, 1)\) 在抛物线 \(C: x^2 = 2py(p > 0)\) 上,过点 \(B(0, -1)\) 的直线交 \(C\)\(P, Q\) 两点,则下列说法中正确的有:

  1. \(C\) 的准线方程为 \(y = -1\)
  2. 直线 \(AB\)\(C\) 相切
  3. \(|OP| \cdot |OQ| > |OA|^2\)
  4. \(|BP| \cdot |BQ| > |BA|^2\)

首先显然得到 \(p = \dfrac12\),进而得到 \(C\) 准线方程为 \(y = -\dfrac14\),选项 (A) 错误。

直线 \(AB\) 方程为 \(y = 2x - 1\),代入抛物线方程得到 \(x^2 = 2x - 1 \iff (x - 1)^2 = 0\),此方程显然仅有 \(x = 1\) 一个实数根,所以选项 (B) 正确。

不妨假设 \(P, Q\) 坐标分别为 \((x_1, x_1^2), (x_2, x_2^2)\)

如果假设 \(PQ\) 斜率为 \(k\),那么 \(PQ\) 方程为 \(y = kx - 1\),代入抛物线方程得到下述二次方程:

\[ x^2 = kx - 1 \iff x^2 - kx + 1 = 0 \]

\(x_1, x_2\) 为上述方程的两根,从而我们有 \(x_1x_2 = 1\)

那么:

\[ \begin{aligned} |OP|^2 \cdot |OQ|^2 &= (x_1^2 + x_1^4)(x_2^2 + x_2^4) \\ &= (x_1^2 + x_1^4)\left(\frac{1}{x_1^2} + \frac{1}{x_1^4}\right) \\ &= 2 + x_1^2 + \frac{1}{x_1^2} \\ &\geq 4 = |OA|^4 \end{aligned} \]

上述等号无法取到,因为 \(x_1 \neq x_2\),从而 \(x_1 \neq \pm 1\)。这说明了选项 (C) 正确。

现在继续计算 (D) 选项,我们有:

\[ \begin{aligned} |BP|^2 \cdot |BQ|^2 &= (x_1^2 + (x_1^2 + 1)^2)(x_2^2 + (x_2^2 + 1)^2) \\ &= (x_1^4 + 3x_1^2 + 1)\left(\frac{1}{x_1^4} + \frac{3}{x_1^2} + 1\right) \\ &= 11 + \left(x_1^4 + \frac{1}{x_1^4}\right) + 6\left(x_1^2 + \frac{1}{x_1^2}\right) \\ &\geq 25 = |BA|^4 \end{aligned} \]

上述等号无法取到,因为 \(x_1 \neq x_2\),从而 \(x_1 \neq \pm 1\)。这说明了选项 (D) 正确。

所以答案为 (BCD)。

Problem 12

已知函数 \(f(x)\) 以及其导函数 \(f'(x)\) 的定义域均为 \(\mathbb R\),记 \(g(x) := f'(x)\)

如果 \(f\left(\dfrac32 - x\right), g(2 + x)\) 均为偶函数,那么下列说法中正确的是:

  1. \(f(0) = 0\)
  2. \(g\left(-\dfrac12\right) = 0\)
  3. \(f(-1) = f(4)\)
  4. \(g(-1) = g(2)\)

首先需要熟知这样的结论,即导函数为偶函数,则原函数为关于某点中心对称,导函数为奇函数,则原函数为偶函数。

由于 \(f(2 + x)\) 的导函数 \(g(2 + x)\) 为偶函数,那么 \(f(2 + x)\) 关于点 \((2, f(2))\) 中心对称。

那么我们可以给出下述推理:

\[ \begin{aligned} f(x) &= f\left(\frac32 - \left(\frac32 - x\right)\right) = f\left(\frac32 + \left(\frac32 - x\right)\right) \\ &= f(3 - x) = f(2 + (1 - x)) = 2f(2) - f(2 - (1 - x)) \\ &= 2f(2) - f(1 + x) \end{aligned} \]

也就得到 \(f(x + 2) = 2f(2) - f(x + 1) = f(x)\)\(f(x)\) 为周期为 \(2\) 的周期函数。

对于选项 (A),如果 \(f(2) \neq 0\),显然可知 \(f(0) \neq 0\)。而这是存在的,取反例 \(f(x) = \sin(\pi x) + 1\) 即可。选项 (A) 错误。

根据 \(f(x)\) 周期显然可知 \(f(2n) = f(2), n \in \mathbb Z\)

同时有 \(f(2n + 1) = f(1) = 2f(2) - f(2) = f(2), n \in \mathbb Z\)。也就是说对所有整数 \(n\),都有 \(f(n) = f(2)\),所以选项 (C) 正确。

此外根据导函数定义(下面所有操作的基础均为导函数的存在性):

\[ g\left(\frac32\right) = \lim_{\delta\to 0}\frac{f\left(\dfrac32 + \delta\right) - f\left(\dfrac32\right)}{\delta} \]

根据 \(f\left(\dfrac32 - x\right)\) 为偶函数,得到:

\[ g\left(\frac32\right) = \lim_{\delta\to 0}\frac{f\left(\dfrac32 - \delta\right) - f\left(\dfrac32\right)}{\delta} = \lim_{\delta\to 0}\frac{f\left(\dfrac32 + \delta\right) - f\left(\dfrac32\right)}{-\delta} \]

上述第二个等号作换元 \(\delta\to-\delta\)。上述两个式子相加得到 \(g\left(\dfrac32\right) = 0\),据 \(g(x)\) 的周期性即可得到 \(g\left(-\dfrac12\right) = 0\)。选项 (B) 正确。

选项 (D) 存在反例 \(f(x) = \sin(\pi x)\),该函数满足所有题设约束,但是不满足 (D) 要求。

所以答案为 (BC)。

填空题

Problem 13

\(\left(1 - \dfrac yx\right)(x + y)^8\) 的展开式中 \(x^2y^6\) 的系数。


最终展开式中 \(x^2y^6\) 项可能的出现方式为 \(1\cdot x^2y^6\) 以及 \(-\dfrac yx \cdot x^3y^5\)。那么最终系数为:

\[ C_8^2 - C_8^3 = \frac{8 \times 7}{2!} - \frac{8 \times 7 \times 6}{3!} = -28 \]

故答案为 \(-28\)

Problem 14

写出与圆 \(x^2 + y^2 = 1\)\((x - 3)^2 + (y - 4)^2 = 16\) 都相切的一条直线的方程。


假设直线方程为 \(ax + by + c = 0\),显然根据直线和圆相切的要求:

\[ \begin{cases} \dfrac{|c|}{\sqrt{a^2 + b^2}} = 1 \\ \dfrac{|3a + 4b + c|}{\sqrt{a^2 + b^2}} = 4 \\ \end{cases} \]

根据第一个方程显然有 \(c \neq 0\),那么记 \(a' := \dfrac ac, b' := \dfrac bc\),上述方程组即:

\[ \begin{cases} \dfrac{1}{\sqrt{a'^2 + b'^2}} = 1 \\ \dfrac{|3a' + 4b' + 1|}{\sqrt{a'^2 + b'^2}} = 4 \\ \end{cases} \Rightarrow \begin{cases} \sqrt{a'^2 + b'^2} = 1 \\ |3a' + 4b' + 1| = 4 \\ \end{cases} \]

分情况讨论,若 \(3a' + 4b' + 1 = 4 \iff 3a' + 4b' = 3\),那么:

\[ b' = \frac{3 - 3a'}{4} \Rightarrow a'^2 + b'^2 = a'^2 + \left(\frac{3 - 3a'}{4}\right)^2 = 1 \]

即:

\[ 25a'^2 - 18a' - 7 = 0 \iff a' \in \left\{1, -\frac{7}{25}\right\} \]

对应的 \(b'\) 分别为 \(0\)\(\dfrac{24}{25}\)

\(3a' + 4b' + 1 = -4 \iff 3a' + 4b' = -5\),那么:

\[ b' = \frac{-5 - 3a'}{4} \Rightarrow a'^2 + b'^2 = a'^2 + \left(\frac{-5 - 3a'}{4}\right)^2 = 1 \]

得到 \(a' = -\dfrac35\),对应的 \(b' = -\dfrac45\)

从而得到答案为 \(x = -1\)\(-7x + 24y + 25 = 0\)\(-3x - 4y + 5 = 0\)

Problem 15

若曲线 \(y = (x + a)e^x\) 有两条过坐标原点的切线,求 \(a\) 的取值范围。


首先:

\[ y' = (x + a + 1)e^x \]

那么曲线在 \(x = x_0\) 处切线方程为:

\[ y = (x_0 + a + 1)e^{x_0}(x - x_0) + (x_0 + a)e^{x_0} \]

该切线过坐标原点等价于:

\[ -x_0(x_0 + a + 1)e^{x_0} + (x_0 + a)e^{x_0} = 0 \iff x_0 + a = x_0(x_0 + a + 1) \]

也就是说切线过坐标原点等价于 \(x_0\) 为下述二次方程的根:

\[ x_0^2 + ax_0 - a = 0 \]

根据题设,上述方程有两个互不相同的实根,即:

\[ \Delta = a^2 + 4a > 0 \iff a \in (-\infty, -4) \cup (0, +\infty) \]

所以答案为 \((-\infty, -4) \cup (0, +\infty)\)

Problem 16

已知椭圆 \(C: \dfrac{x^2}{a^2} + \dfrac{y^2}{b^2} = 1(a > b > 0)\)\(C\) 的上顶点为 \(A\),两个焦点为 \(F_1, F_2\),离心率为 \(\dfrac12\)。过 \(F_1\) 且垂直于 \(AF_2\) 的直线交 \(C\)\(D, E\) 两点,\(|DE| = 6\),求 \(\triangle ADE\) 的周长。


根据离心率,不妨假设 \(a = 2t, b = \sqrt3 t, c = t(t > 0)\)。那么 \(A\) 坐标为 \((0, \sqrt3 t)\)\(F_1, F_2\) 坐标为 \((\pm t, 0)\)

那么 \(AF_2\) 斜率为 \(-\sqrt 3\),得知 \(DE\) 的斜率为 \(\dfrac{1}{\sqrt 3}\),那么 \(DE\) 方程为 \(x = \sqrt3y - t\)。代入椭圆方程得到:

\[ \dfrac{(\sqrt3y - t)^2}{4t^2} + \dfrac{y^2}{3t^2} = 1 \iff 13y^2 - 6\sqrt3 yt - 9t^2 = 0 \]

假设上述关于 \(y\) 的方程的两个根为 \(y_1, y_2\),那么 \(y_1 + y_2 = \dfrac{6\sqrt3 t}{13}, y_1y_2 = -\dfrac{9}{13}t^2\)

我们显然知道:

\[ \begin{aligned} |DE|^2 &= 4(y_1 - y_2)^2 = 4((y_1 + y_2)^2 - 4y_1y_2) \\ &= \left(\frac{48t}{13}\right)^2 = 36 \end{aligned} \]

得到 \(t = \dfrac{13}{8}\)

另外,直接求解方程得到:

\[ y_{1, 2} = \frac{3\sqrt 3 \pm 12}{13}t = \frac{3\sqrt 3}{8} \pm \frac32 \]

从而得到 \(D, E\) 的坐标为:

\[ \left(\frac{3\sqrt 3}{2} - \frac12, \frac{3\sqrt 3}{8} + \frac32\right), \left(-\frac{3\sqrt 3}{2} - \frac12, \frac{3\sqrt 3}{8} - \frac32\right) \]

考虑到 \(A\) 坐标为 \(\left(0, \dfrac{13\sqrt 3}{8}\right)\),得到:

\[ \begin{aligned} |AD|^2 &= \left(\frac{3\sqrt 3}{2} - \frac12\right)^2 + \left(\frac{3\sqrt 3}{8} + \frac32 - \dfrac{13\sqrt 3}{8}\right)^2 \\ &= \frac{223 - 84\sqrt 3}{16} \\ |BD|^2 &= \left(-\frac{3\sqrt 3}{2} - \frac12\right)^2 + \left(\frac{3\sqrt 3}{8} - \frac32 - \dfrac{13\sqrt 3}{8}\right)^2 \\ &= \frac{223 + 84\sqrt 3}{16} \\ \end{aligned} \]

不难发现 \((14 \pm 3\sqrt 3)^2 = 223 \pm 84\sqrt 3\),所以:

\[ |AD| + |BD| = \frac{14 - 3\sqrt 3}{4} + \frac{14 + 3\sqrt 3}{4} = 7 \]

从而需要求的三角形周长为 \(13\)。即最后的答案为 \(13\)


【另解】我自己也是算完之后才回想起来有这么一个简单的结论我忘了,离心率为 \(\dfrac12\) 的椭圆,其上顶点和两个焦点构成等边三角形。

这个事实意味着直线 \(DE\) 不仅垂直于 \(AF_2\),而且是垂直平分 \(AF_2\)。也就是说 \(\triangle ADE\) 全等于 \(\triangle DEF_2\)。那么其周长:

\[ C_{\triangle DEF_2} = (|DF_1| + |DF_2|) + (|EF_1| + |EF_2|) = 4a = 13 \]

解答题

这次解答题的顺序是数列、解三角形、立体几何、统计、解析几何、导数,中规中矩的顺序。

Problem 17

\(S_n\) 为数列 \(\{a_n\}\) 的前 \(n\) 项和,已知 \(a_1 = 1\),且数列 \(\left\{\dfrac{S_n}{a_n}\right\}\) 是公差为 \(\dfrac13\) 的等差数列。

  1. \(\{a_n\}\) 的通项公式。

  2. 求证:

\[ \sum_{k = 1}^n \frac{1}{a_k} < 2, n \in \mathbb N \]


【第一问】根据数列 \(\left\{\dfrac{S_n}{a_n}\right\}\) 是公差为 \(\dfrac13\) 的等差数列,并且我们能得到其首项为:

\[ \frac{S_1}{a_1} = 1 \]

那么我们得到其通项公式为:

\[ \frac{S_n}{a_n} = \frac13 n + \frac23 \iff 3S_n = (n + 2)a_n \]

\(n + 1\) 代替 \(n\),得到等式 \(3S_{n + 1} = (n + 3)a_{n + 1}\),上述两式相减:

\[ 3(S_{n + 1} - S_n) = (n + 3)a_{n + 1} - (n + 2)a_n \iff (n + 2)a_n = na_{n + 1} \]

也就是下述关系:

\[ \frac{a_{n + 1}}{a_n} = \frac{n + 2}{n}, n \in \mathbb N \]

那么对于 \(n \geq 2\)

\[ \begin{aligned} a_n &= \frac{a_n}{a_{n - 1}} \cdot \frac{a_{n - 1}}{a_{n - 2}} \cdot \cdots \cdot \frac{a_2}{a_1} = \frac{n + 1}{n - 1} \cdot \frac{n}{n - 2} \cdot \cdots \cdot \frac{3}{1} \\ &= \frac{n(n + 1)}{2} \end{aligned} \]

经检验,\(n = 1\) 时上述通项公式也成立,所以数列 \(\{a_n\}\) 的通项公式为:

\[ a_n = \frac{n(n + 1)}{2}, n \in \mathbb N \]

【第二问】可以计算得到:

\[ \begin{aligned} \sum_{k = 1}^n \frac{1}{a_k} = \sum_{k = 1}^n \frac{2}{k(k + 1)} = 2\sum_{k = 1}^n \left(\frac{1}{k} - \frac{1}{k + 1}\right) = 2\left(1 - \frac{1}{n + 1}\right) < 2 \end{aligned} \]

Problem 18

\(\triangle ABC\) 的内角 \(A, B, C\) 的对边分别为 \(a, b, c\)。已知下述关系:

\[ \frac{\cos A}{1 + \sin A} = \frac{\sin 2B}{1 + \cos 2B} \]

  1. \(C = \dfrac{2\pi}{3}\),求 \(B\)

  2. \(\dfrac{a^2 + b^2}{c^2}\) 的最小值


【第一问】根据二倍角公式有:

\[ \frac{\sin 2B}{1 + \cos 2B} = \frac{2\sin B\cos B}{1 + (2\cos^2 B - 1)} = \frac{\sin B}{\cos B} \]

代入已知等式:

\[ \frac{\cos A}{1 + \sin A} = \frac{\sin B}{\cos B} \]

整理得到:

\[ \cos A\cos B = \sin B + \sin A\sin B \Rightarrow \sin B = \cos A\cos B - \sin A\sin B = \cos(A + B) \]

根据三角形内角和,有 \(A + B + C = \pi \Rightarrow A + B = \pi - C\),即:

\[ \sin B = \cos(\pi - C) = -\cos C \]

已知 \(C = \dfrac{2\pi}{3}\),所以:

\[ \sin B = -\cos\frac{2\pi}{3} = \frac12 \]

由于 \(B\) 位于三角形内,所以 \(B \in (0, \pi)\)

另外,考虑到 \(C = \dfrac{2\pi}{3}\),若 \(B > \dfrac\pi2\),则 \(A = \pi - C - B < 0\),不符合要求,所以 \(B < \dfrac\pi2\)

从而 \(B = \dfrac\pi6\)

【第二问】根据上一问:

\[ \cos(A + B) = \sin B = \cos\left(\frac\pi2 - B\right) \]

由于 \(A + B, \dfrac\pi2 - B \in (0, \pi)\),而余弦函数在这一个区间单调,所以:

\[ A + B = \dfrac\pi2 - B \iff A + 2B = \frac\pi2 \]

根据正弦定理:

\[ \frac{a^2 + b^2}{c^2} = \frac{\sin^2A + \sin^2B}{\sin^2C} \]

使用 \(-\cos C = \sin B\) 替换,并代入 \(A + 2B = \dfrac\pi2\)

\[ \begin{aligned} \frac{a^2 + b^2}{c^2} &= \frac{\sin^2A + \sin^2B}{\sin^2C} = \frac{\sin^2\left(\dfrac\pi2 - 2B\right) + \sin^2B}{1 - \cos^2C} \\ &= \frac{\cos^2 2B + \sin^2B}{1 - \sin^2B} = \frac{(2\cos^2 B - 1)^2 + (1 - \cos^2B)}{\cos^2B} \\ &= 4\cos^2B + \frac{2}{\cos^2B} - 5 \geq 4\sqrt2 - 5 \end{aligned} \]

等号成立当且仅当 \(\cos B = \dfrac{1}{\sqrt[4]{2}}\)

\(\dfrac{a^2 + b^2}{c^2}\) 的最小值为 \(4\sqrt2 - 5\)

Problem 19

已知直三棱柱 \(ABC-A_1B_1C_1\) 的体积为 \(4\)\(\triangle A_1BC\) 的面积为 \(2\sqrt 2\)

  1. \(A\) 到平面 \(A_1BC\) 的距离

  2. \(D\)\(A_1C\) 的中点,已知 \(AA_1 = AB\) 且平面 \(A_1BC\) 垂直于平面 \(ABB_1A_1\),求二面角 \(A-BD-C\) 的正弦值。


【第一问】考虑三棱锥 \(A_1-ABC\),其和直三棱柱 \(ABC-A_1B_1C_1\) 有相同的底面 \(\triangle ABC\) 且有相同的高 \(AA_1\),所以其体积为直三棱柱 \(ABC-A_1B_1C_1\) 的三分之一,即:

\[ V_{A_1-ABC} = \frac13 \times 4 = \frac43 \]

如果记 \(A\) 到平面 \(A_1BC\) 的距离为 \(h\)\(\triangle A_1BC\) 的面积为 \(S\),那么 \(V_{A_1-ABC} = \dfrac13 Sh\)。从而得到:

\[ \dfrac13 Sh = \frac43 \Rightarrow Sh = 4 \]

由题设有 \(S = 2\sqrt 2\),所以 \(A\) 到平面 \(A_1BC\) 的距离为:

\[ h = \frac{4}{2\sqrt 2} = \sqrt 2 \]

【第二问】

在直三棱柱 \(ABC-A_1B_1C_1\) 中,我们有 \(BB_1\) 垂直于平面 \(ABC\),考虑到 \(BC \subset\) 平面 \(ABC\),所以 \(BB_1 \bot BC\)

另一方面,直三棱柱 \(ABC-A_1B_1C_1\) 中侧面 \(ABB_1A_1\) 为长方形,而 \(AA_1 = AB\),所以该侧面为正方形,进而 \(AB_1 \bot A_1B\)

由于平面 \(A_1BC\) 垂直于平面 \(ABB_1A_1\) 且两平面交于 \(A_1B\),考虑到 \(AB_1 \subset\) 平面 \(ABB_1A_1\),所以 \(AB_1\) 垂直于平面 \(A_1BC\)。而又由于 \(BC \subset\) 平面 \(ABC\),所以 \(AB_1 \bot BC\)

因为 \(AB_1, BB_1\) 都是平面 \(ABB_1A_1\) 内的直线且两者相交于 \(B_1\),而直线 \(BC\) 与两者均垂直,所以 \(BC\) 垂直于平面 \(ABB_1A_1\)。考虑到 \(AB \subset\) 平面 \(ABB_1A_1\),所以 \(AB \bot BC\)。同理依然有 \(A_1B \bot BC\)

也就是说 \(\triangle ABC\) 为以 \(AC\) 为斜边的直角三角形,\(\triangle A_1BC\) 为以 \(A_1C\) 为斜边的直角三角形。设 \(AA_1 = AB = a, BC = b\),那么 \(A_1B = \sqrt2 a\)\(\triangle ABC\) 面积为 \(\dfrac12 ab\)\(\triangle A_1BC\) 面积为 \(\dfrac{\sqrt 2}{2}ab = 2\sqrt 2\)

那么直三棱柱 \(ABC-A_1B_1C_1\) 的体积为 \(a\cdot \dfrac12 ab = \dfrac12 a^2b = 4\)

从而得到 \(a = b = 2\)

上述论述中我们已经说明了 \(AB, B_1B, BC\) 两两垂直,所以以 \(B\) 为坐标原点,\(BC, BA, BB_1\) 分别为 \(x, y, z\) 轴建立空间直角坐标系。

那么 \(A\) 坐标为 \((0, 2, 0)\)\(A_1\) 坐标为 \((0, 2, 2)\)\(C\) 坐标为 \((2, 0, 0)\)

由于 \(D\)\(A_1C\) 中点,所以其坐标为 \((1, 1, 1)\)

那么 \(\vec{BA} = (0, 2, 0), \vec{BD} = (1, 1, 1), \vec{BC} = (2, 0, 0)\)

假设平面 \(ABD\) 的法向量为 \(\boldsymbol{n} = (x_1, y_1, z_1)\),根据 \(\boldsymbol{n} \bot \vec{BA}, \vec{BD}\) 得到:

\[ \begin{cases} x_1 + y_1 + z_1 = 0 \\ 2y_1 = 0 \end{cases} \]

不妨取 \(\boldsymbol{n} = (1, 0, -1)\)。同理可以取平面 \(CBD\) 的法向量 \(\boldsymbol{m} = (0, 1, -1)\)

记欲求二面角大小为 \(\theta\),可以得到二面角 \(A-BD-C\) 的余弦值绝对值为:

\[ |\cos\langle\boldsymbol{n}, \boldsymbol{m}\rangle| = \left|\frac{\boldsymbol{n} \cdot \boldsymbol{m}}{|\boldsymbol{n}| \cdot |\boldsymbol{m}|}\right| = \frac{1}{2} \]

从而得到该二面角的正弦值为:

\[ \sin\theta = \sqrt{1 - |\cos\theta|^2} = \frac{\sqrt 3}{2} \]

Problem 20

概率统计相关问题。医疗团队针对某一种疾病和个人卫生的关系做调查,抽样 \(100\) 名该病患者构成病例组和 \(100\) 名未患有该病的健康人构成对照组,调查他们的个人卫生,得到下述数据:

个人卫生不够良好 个人卫生良好
病例组 \(40\) \(60\)
对照组 \(10\) \(90\)
  1. 能否有 \(99\%\) 的把握认为患该病的群体和未患该病的群体个人卫生情况有差异

  2. 从该地区人群中任选一人,记 \(A\) 表示事件“选到的人卫生习惯不够良好”,\(B\) 表示事件“选到的人患有该疾病”。记指标 \(R\) 为:

\[ R = \frac{P(B \mid A)}{P(\overline B \mid A)} \cdot \frac{P(\overline B | \overline A)}{P(B \mid \overline A)} \]

  1. 证明:

\[ R = \frac{P(A \mid B)}{P(\overline A \mid B)} \cdot \frac{P(\overline A | \overline B)}{P(A \mid \overline B)} \]

  1. 利用调查数据估计 \(P(A \mid B), P(A \mid \overline B)\),并根据上述结论估计 \(R\) 值。

【第一问】直接使用卡方检验:

\[ K^2 = \frac{n(ad - bc)^2}{(a + b)(c + d)(a + c)(b + d)} = \frac{200 \times (40 \times 90 - 10 \times 60)^2}{100 \times 100 \times 50 \times 150} = 24 > 6.635 \]

可知我们有 \(99\%\) 的把握认为患该病的群体和未患该病的群体个人卫生情况有差异。

【第二问】我们首先证明第一小问的等式:

根据 \(R\) 的定义和条件概率的定义:

\[ \begin{aligned} R &= \frac{P(B \mid A)}{P(\overline B \mid A)} \cdot \frac{P(\overline B | \overline A)}{P(B \mid \overline A)} = \frac{\dfrac{P(AB)}{P(A)} \cdot \dfrac{P(\overline A\overline B)}{P(\overline A)}}{\dfrac{P(A\overline B)}{P(A)} \cdot \dfrac{P(\overline AB)}{P(\overline A)}} \\ &= \frac{P(AB)P(\overline A\overline B)}{P(A\overline B)P(\overline AB)} \\ &= \frac{\dfrac{P(AB)}{P(B)} \cdot \dfrac{P(\overline A\overline B)}{P(\overline B)}}{\dfrac{P(A\overline B)}{P(\overline B)} \cdot \dfrac{P(\overline AB)}{P(B)}} = \frac{P(A \mid B)}{P(\overline A \mid B)} \cdot \frac{P(\overline A | \overline B)}{P(A \mid \overline B)} \end{aligned} \]

容易根据调查数据估计出:

\[ P(A \mid B) = 0.4, P(\overline A \mid B) = 0.6, P(\overline A | \overline B) = 0.9, P(A \mid \overline B) = 0.1 \]

此时 \(R = 6\)

Problem 21

已知点 \(A(2, 1)\) 在双曲线 \(C: \dfrac{x^2}{a^2} - \dfrac{y^2}{a^2 - 1} = 1(a > 1)\) 上,直线 \(l\)\(C\)\(P, Q\) 两点,直线 \(AP, AQ\) 的斜率之和为 \(0\)

  1. \(l\) 的斜率

  2. \(\tan\angle PAQ = 2\sqrt 2\),求 \(\triangle PAQ\) 的面积


【第一问】由于 \(A(2, 1)\) 在双曲线 \(C\) 上,所以:

\[ \dfrac{4}{a^2} - \dfrac{1}{a^2 - 1} = 1 \Rightarrow a^2 = 2 \]

不妨记 \(AP\) 的斜率为 \(k\),那么 \(AQ\) 的斜率为 \(-k\)。从而 \(AP\) 的方程为 \(y = k(x - 2) + 1\),将其代入双曲线方程得到:

\[ \dfrac{x^2}{2} - (kx - 2k + 1)^2 = 1 \]

整理为:

\[ \left(\frac{1}{2} - k^2\right)x^2 + 2(2k - 1)kx - (1 + (2k - 1)^2) = 0 \]

由于 \(A\) 为双曲线和 \(AP\) 的交点之一,所以 \(x = 2\) 为其一根,另一根为 \(P\) 的横坐标。

上述方程的两根之积为:

\[ -\frac{1 + (2k - 1)^2}{\dfrac{1}{2} - k^2} = -\frac{2 + 2(2k - 1)^2}{1 - 2k^2} \]

得知其横坐标即可计算其纵坐标,所以 \(P\) 的坐标为:

\[ \left(-\frac{1 + (2k - 1)^2}{1 - 2k^2}, \frac{2k^2 - 4k + 1}{1 - 2k^2}\right) \]

\(k\) 替换为 \(-k\) 得到 \(Q\) 的坐标:

\[ \left(-\frac{1 + (2k + 1)^2}{1 - 2k^2}, \frac{2k^2 + 4k + 1}{1 - 2k^2}\right) \]

也就是说 \(l\) 的斜率为:

\[ k_l = \frac{-\dfrac{1 + (2k + 1)^2}{1 - 2k^2} + \dfrac{1 + (2k - 1)^2}{1 - 2k^2}}{\dfrac{2k^2 + 4k + 1}{1 - 2k^2} - \dfrac{2k^2 - 4k + 1}{1 - 2k^2}} = -1 \]

【第二问】由于 \(\tan\angle PAQ > 0\),所以其为锐角。我们可以知道 \(\tan\dfrac12\angle PAQ = \dfrac1k\),这里 \(k\)\(AP, AQ\) 斜率的绝对值。从而:

\[ \tan\angle PAQ = \frac{2\tan\dfrac12\angle PAQ}{1 - \tan^2\dfrac12\angle PAQ} = \frac{2 / k}{1 - 1 / k^2} = 2\sqrt 2 \]

\(k > 0\) 得到 \(k = \sqrt 2\)

从而根据上一问的结果,我们得到:

\[ P\left(\frac{10 - 4\sqrt 2}{3}, \frac{4\sqrt 2 - 5}{3}\right), Q\left(\frac{10 + 4\sqrt 2}{3}, \frac{-4\sqrt 2 - 5}{3}\right) \]

此时 \(PQ\) 方程为 \(y = -x + \dfrac53\)\(A\) 到此的距离为:

\[ d = \frac{|2 + 1 - 5 / 3|}{\sqrt 2} = \frac{2\sqrt 2}{3} \]

而:

\[ |PQ| = \sqrt 2 \left(\frac{10 + 4\sqrt 2}{3} - \frac{10 - 4\sqrt 2}{3}\right) = \frac{16}{3} \]

所以需要求的面积为:

\[ S = \frac12 |PQ| \cdot d = \frac{16\sqrt 2}{9} \]

Problem 22

已知函数 \(f(x) = e^x - ax\) 与函数 \(g(x) = ax - \ln x\) 有相同的最小值。

  1. \(a\)

  2. 证明存在实数 \(b\),令直线 \(y = b\) 和曲线 \(y = f(x), y = g(x)\) 共计有三个不同的交点,且这三个交点的横坐标从小到大排列构成等差数列。


【第一问】显然:

\[ f'(x) = e^x - a, g'(x) = a - \frac1x \]

由于这两个函数有相同的最小值,所以:

\[ f(\ln a) = a - a\ln a = 1 + \ln a = g\left(\frac1a\right) \]

也就是说:

\[ \ln a - \frac{a - 1}{a + 1} = 0 \]

另外,由最小值的存在性显然有 \(a > 0\)。考虑函数:

\[ \phi(x) := \ln x - \frac{x - 1}{x + 1}, x > 0 \]

其有:

\[ \phi'(x) = \frac1x - \frac{2}{(x + 1)^2} = \frac{x^2 + 1}{x(x + 1)^2} > 0 \]

\(\phi(a) = \phi(1) = 0\) 得到 \(a = 1\)

【第二问】首先证明,\(f(x) - g(x) = 0\)\((0, +\infty)\) 上有唯一的解 \(x_0\)

定义函数 \(h(x) := f(x) - g(x)\)

根据上一问,得知 \(f(x)\)\((0, +\infty)\) 上递增,\(g(x)\)\((0, 1)\) 上递减,这说明 \(h(x)\)\((0, 1)\) 上递增。

另外:

\[ h'(x) = e^x + \frac1x - 2 > e - 2 > 0, x > 1 \]

所以 \(h(x)\)\((1, +\infty)\) 上递增,即 \(h(x)\)\((0, +\infty)\) 上递增。

此外:

\[ h(1) = e - 2 > 0 \]

另外:

\[ g(e^{1 - e}) = e^{1 - e} - (1 - e) > e - 1 = f(1) > f(e^{1 - e}) \]

\(h(e^{1 - e}) < 0\)

这说明了 \(x_0\) 唯一存在于区间 \((e^{1 - e}, 1)\) 上。

下面证明 \(b = f(x_0) = g(x_0)\) 满足条件。

在此之前,我们需要知道由于 \(0 < x_0 < 1\),所以 \(1 = f(0) < b = f(x_0) < e - 1 = f(1)\)

首先证明 \(f(x) = b\) 有除 \(x_0\) 外,还具有一根 \(x_1 \in (-\infty, 0)\)。由于 \(f(x)\)\((-\infty, 0)\) 上递减,并且 \(f(0) < b\),且 \(f(1 - e) = e^{1 - e} + (e - 1) > e - 1 > b\)

这说明了 \(x_1\) 唯一存在于 \((1 - e, 0)\) 上。

再证明 \(g(x) = b\) 有除 \(x_0\) 外,还具有一根 \(x_2 \in (1, +\infty)\)。由于 \(g(x)\)\((1, +\infty)\) 上递增,并且 \(g(1) = 1 < b\),且 \(g(e) = e - 1 > b\)

这说明了 \(x_2\) 唯一存在于 \((1, e)\) 上。

也就是说直线 \(y = b\) 和曲线 \(y = f(x), y = g(x)\) 共计有三个不同的交点,其横坐标从小到大为 \(x_1, x_0, x_2\)

现在观察下述:

\[ \begin{cases} f(\ln x_0) = e^{\ln x_0} - \ln x_0 = x_0 - \ln x_0 = g(x_0) = b \\ g(e^{x_0}) = e^{x_0} - \ln e^{x_0} = e^{x_0} - x_0 = f(x_0) = b \\ \end{cases} \]

这说明 \(\ln x_0, e^{x_0}\) 也是直线 \(y = b\) 和曲线 \(y = f(x), y = g(x)\) 交点的横坐标,但我们已经证明了一共仅有三个交点,这只能说明 \(x_1 = \ln x_0, x_2 = e^{x_0}\)

由于 \(f(x_0) = g(x_0)\),所以 \(e^{x_0} + \ln x_0 = x_1 + x_2 = 2x_0\)

所以这三个横坐标从小到大排列为等差数列,证明完毕。

后记

卷子做完了,Holder 也已经明白了我已经没有高中时候那种手算能力了。

不少人说这份卷子很难,其实我看真正算得上是难题的几乎没有,几乎每道题都有很自然的思考角度和解决方式。

但问题就是现在大部分的高考备考流于做模板题,练死思路,结果就是这次高考连稍微难点的解三角形、立体几何都做不出来。

复习不够全面,可能连条件概率的定义、棱台的体积公式都会忘记。


这份卷子只不过是把平时用来送分的解答题前几条,数列、解三角形、立体几何稍微出得有了点新意,稍微加了点门槛。卷子其他地方平心而论,概率与统计只要记得条件概率定义五分钟就能写完,解析几何是考烂了的二级结论,选择填空也乏善可陈。但就是这样,就能把平时的中档生直接卡住。

而且这份卷子的特征就是,思路的高下直接影响分数。想到了妙手,就能一分钟内看出答案,想不到,那就和我做填空题最后一问一样算到起飞还得不到答案。一旦陷入了这种状况,如果不及时调整思路,可能影响的是后面的解题状态。

说明死方法没用了,脑子要灵活。总而言之,是一份好卷子,但是被骂也是理所应当。